A train ticket in a certain city is 3.00. People eho use the train also have the option of purchasing a frequent rider pass for 18.75 each month. With the pass, each ticket costs only 2.25. Determine the number of times in a month the train must be used so that the total monthly cost without the pass is the same as the total monthly cost with the pass.

Answers

Answer 1

The number of times for which the train must be used so that the monthly cost without the pass is same as with the pass is; 6.25 times.

How to solve Word Problems using algebraic expressions.

It follows from the task content that the regular train ticket costs, 3.00.

Therefore, the cost for travelling x times with the month is; 3x.

On this note, for the total monthly cost without the pass to be same as that with the pass; the equation which must hold is;

3x = 18.75

x = 18.75/3

x = 6.25 times.

Therefore, after taking the train 6.25 times, the cost without the pass is same as that with the pass.

Read more on word problems;

https://brainly.com/question/21405634

#SPJ1


Related Questions

9 The 11th term of an A.P. is -31 and
21st term is -71, find the (a) 1st term
(b) common difference (c) 15th term

Answers

(a) The 1st term of the arithmetic progression is 9

(b) The common difference of the arithmetic progression is -4

(c) The 15th term of the arithmetic progression is -47

Calculating the terms of an Arithmetic Progression

The nth term of an arithmetic progression is given by

aₙ = a + (n - 1)d

Where a is the first term

and d is the common difference

From the given information,

a₁₁ = -31

and

a₂₁ = -71

But

a₁₁ = a + 10d

and

a₂₁ = a + 20d

Thus,

a + 10d = -31          ------------ (1)

a + 20d = -71         ------------ (2)

Subtract equation (1) from equation (2)

a + 20d = -71

a + 10d = -31

---------------------------

10d = -40

d = -40/10

d = -4

Substitute the value of d into equation (1)

a + 10d = -31

a + 10(-4) = -31

a - 40 = -31

a = -31 + 40

a = 9

Also,

a₁₅ = a + 14d

Substitute the values of a and d

a₁₅ = 9 + 14(-4)

a₁₅ = 9 - 56

a₁₅ = -47

Hence, the first term is 9, the common difference is -4 and the 15th term is -47

Learn more on Arithmetic progression here: https://brainly.com/question/24989563

#SPJ1

Look at this diagram:If KM and NP are parallel lines and m

Answers

Answer

Angle NOL = 70°

Explanation

Alternate angles are angles that are in opposite positions relative to a transversal intersecting two lines. If the two lines are parallel to each other, then alternate angles are equal.

We can see that Angle MLO and Angle NOL are alternate angles.

And since we have been told that KM and NP are parallel lines,

Angle NOL = Angle MLO = 70°

Hope this Helps!!!

893 is 94% of what amount

Answers

4,700 is correct for this question

Fun times Amusement Park Charges a $7 admission fee plus $1.75 per ride. What is the equation, in intercept form for calculating the total cost, C, of going to the park and riding r rides?1. C = 1.75 + 7r2. C = 1.75r3. C = 7r4. C = 7 + 1.75r

Answers

The general form of an equation in intercetp form is:

[tex]y=mx+b[/tex]

Where m is the change of y in function of x and b is the value of b when x is 0

In this situation you have:

y=C

x=r

m= 1.75 ( C increase 1.75 each r)

b= 7 (When there is not ride r=0 the admission fee is 7)

Then, you have the equation:[tex]C=1.75r+7[/tex]or C = 7 + 1.75r

given angle 1 is congruent toangle 3 and angle 12 is congruent to angle 8 prove l is parallel to m

Answers

Given that;

[tex]\angle1\cong\angle3,\angle12\cong\angle8[/tex]

Line a and b are two straight lines cut by two transversal lines l and m.

The tranversal line l shows that;

[tex]\begin{gathered} \angle8\cong\angle6 \\ \end{gathered}[/tex]

But also;

[tex]\angle8\cong\angle12[/tex]

Thus,

[tex]\angle6\cong\angle12[/tex]

Then, if two lines are cut by a transversal so the corresponding angles are congruent, then the lines are parallel.

Thus, line l is parallel to m

Question 10 !!
Help me please

Answers

letter A = C

letter B = B

Letter C= B

Answer:

(a) Figure C

(b) Figures B and C

(c) Figures B and C

Step-by-step explanation:

Figure A does not apply to any of the 3 questions

Hope this helps

Please help, i really need help

Answers

Answer: m<1 = 146 degrees, m<3 =146 degrees, m<4 = 34 degrees

Step-by-step explanation:

Since m<2 and m<4 are vertical angles, they will be equal to each other.

By using straight lines, we can easily find out that m<1 = 180-34 = 146

Since m<1 and m<3 are vertical angles, they are equal to each other.

Hope this helped :))

Angle 4: is 34 degrees since angle 2 and angle 4 are vertical angles and congruent (equal)
Angle 1: is 146 degrees. Since angle 4 and angle 1 are linear angles they are equal to 180. So subtract 180 and 34 to get 146
Angle 3: is 146 because angles 1 and 3 are vertical and congruent.
Hope this helps :)

The dot plot shows predictions for the winning time in the​ 200-meter sprint. The winner finished the race in 22.3 seconds. What is the greatest percent error among the​ predictions?

Answers

2.69%, is the required maximum percent error among the predictions.

Given,

The predicted winning time in 200 meter sprint shows in the dot plot

The winner finished the race in 22.3 seconds

We have to find the greatest percent error among the predictions;

Here,

The actual time to complete is 22.3 seconds.

22.9 seconds is the prediction that is the most off.

The difference is, 22.9 - 22.3 = 0.6 seconds

To obtain a prediction, divide this by the actual time value.

= 0.6/22.3 = 0.0269 = 2.69

Therefore, 2.69%, is the required maximum percent error among the predictions.

Learn more about predictions here;

https://brainly.com/question/14946661

#SPJ1

The table shows the cumulative number of minutes Alice practices clarinet for the first part of the school year:The table shows the cumulative number of minutes Alice practices clarinet for the first part of the school year:

Answers

The correct option regarding the scale and the origin of the graph are as follows:

D.

x-axis scale: 1 unit = 1 week

y-axis scale: 1 unit = 150 minutes

origin: (0 weeks, 0 minutes)

Scale and origin

The scale should be chosen focusing on improving the readability of the data-set by the reader, while the origin should be chosen according to the values assumed by the variables.

In the context of this problem, the values of x, in weeks, are:

2, 3, 4, 5, 6, 7, 8.

They increase by one, hence the scale of x should be of 1 unit = 1 week.

The values of y, in minutes are given as follows:

300, 450, 600, 750, 900, 1050, 1200.

They increase by 150, hence the scale of y should be of 1 unit = 150 minutes, which is the rate of change of the problem.

As the measures are both positive values, the origin should be of (0,0), hence the correct option for the scales and the origin is option D.

Complete problem

The table is:

Weeks Minutes

2 300

3 450

4 600

5 750

6 900

7 1,050

8 1,200

The options are:

A. x-axis scale: 1 unit = 2 weeks

y-axis scale: 1 unit = 50 minutes

origin: (0 weeks, 0 minutes)

B. x-axis scale: 1 unit = 2 weeks

y-axis scale: 1 unit = 150 minutes

origin: (2 weeks, 300 minutes)

C. x-axis scale: 1 unit = 1 week

y-axis scale: 1 unit = 50 minutes

origin: (2 weeks, 300 minutes)

D. x-axis scale: 1 unit = 1 week

y-axis scale: 1 unit = 150 minutes

origin: (0 weeks, 0 minutes)

Learn moire about scales at https://brainly.com/question/16355151

#SPJ1

a company buys equal numbers of two different card forms. it utilizes 4/5 of one kind and 6/7 of the other. what fraction of the total number is unused?

Answers

12/35 fraction of the total number is unused from two different cards.

What is a fraction?

A fraction is written in the form of p/q, where q ≠ 0.

Fractions are of two types they are proper fractions in which the numerator is smaller than the denominator and improper fractions where the numerator is greater than the denominator.

Assuming the total first kind of card form is 1 and the total second kind of card form is also one.

Given, a company buys equal numbers of two different card forms. it utilizes 4/5 of one kind and 6/7 of the other.

∴ The total unused card form is,

= (1 + 1) - (4/5 + 6/7).

= 2 - (28 + 30)/35.

= 2 - 58/35.

= (70 - 58)/35.

= 12/35.

learn more about fractions here :

https://brainly.com/question/10354322

#SPJ1

An exterior angle of a rectangle polygon cannot have the measure

Answers

The sum of the measures of an exterior angle of a polygon is 360°.

If the given angle divides 360 evenly, then it can be a measure of an exterior angle of a polygon. If otherwise, then it cannot be.

[tex]\begin{gathered} 360\div30=12 \\ 360\div50=7.2 \\ 360\div120=3 \\ 360\div90=4 \\ 360\div40=9 \end{gathered}[/tex]

Out of the given angles, only 50 does not divide 360 evenly. Therefore, a regular polygon cannot have an exterior angle measuring 50°. (Option B)

An adult elephant weighs 2 2/5 tons. An adult hippo weighs 1 4/5 tons. How much will the animals weigh together?

Answers

Answer: If you want the result as a fraction it should be 3.7/5. but if you want a whole number it's 4.2. if you want a mixed number its 4 1/5

Step-by-step explanation:

Claire scheduled a taxi to pick her up at her house. The taxi service says it will take 15 minutes to get to her house based on normal traffic routines. However, due to either no traffic or more traffic than normal, the time could vary by as much as 4 minutes. Which inequality and solution show the range of possible minutes, t, it will take for the taxi to get to Claire's house?

Answers

The inequality would be 15 ≤ t ≤ 19 showing the range of possible minutes it will take for the taxi to get to Claire's house.

What is inequality?

Inequality is defined as mathematical statements that have a minimum of two terms containing variables or numbers that are not equal.

The time could vary by as much as 4 minutes.

Let t be the range of possible minutes

The taxi service says it will take 15 minutes to get to her house based on normal traffic routines.

So the inequality as 15 ≤ t

The time could vary by as much as 4 minutes.

So the inequality as 15 ≤ t ≤ 15 + 4

⇒  15 ≤ t ≤ 19

Therefore, the inequality would be 15 ≤ t ≤ 19 showing the range of possible minutes.

Learn more about the inequalities here:

brainly.com/question/20383699

#SPJ1

Find the inverse of the given function.
5. f= {(1,3), (2,-5), (3,6)} ​

Answers

Check the picture below.

For the function f(x)= x^2+4x-1, what is the range of f (x) for the domain {-2,0,1}?

Answers

The range of the given function is {-5,-1,4} which is the B option.

Given function:-

[tex]f(x) = x^2+4x-1[/tex]

Domain = {-2,0,1}

We have to find the range of the given function for the given domain.

Putting x = -2 in the given function, we get,

[tex]f(-2) = (-2)^2+4(-2)-1[/tex]

f(-2) = 4 - 8 - 1 = -5

Putting x = 0 in the given function, we get,

[tex]f(0) = (0)^2+4(0)-1[/tex]

f(0) = 0 + 0 -1 = -1

Putting x = 1 in the given function, we get,

[tex]f(1) = (1)^2+4(1)-1[/tex]

f(1) = 1 + 4 - 1 = 4

Hence, the range of the given function is {-5,-1,4}.

To learn more about range, here:-

https://brainly.com/question/28135761

#SPJ1

Which brand of granola typically weighs more?

Brand A bags typically weigh more because the median of brand A is higher than that of brand B.
Brand B bags typically weigh more than brand A bags because there is a high outlier at 52.5.
Brand A bags typically weigh more than brand B bags because there are no outliers in the distribution.
Brand B bags typically weigh more because the range of weights is higher than that of brand A.

Answers

Brand A bags typically weigh more because the median of brand A is higher than that of brand B and is denoted as option A.

What is Median?

This is referred to as the middle value which separates the higher half from the lower half of a data or distribution. This is calculated by first ordering the numbers which could be from the lowest to highest or vice versa.

A data sample which has a higher figure as the median means that it has higher values present in the data distribution. This term provides an insight on certain properties of the values of the data which are given.

In this scenario, the median of brand A is higher than that of brand B which means that it has more weight and is therefore the reason why option A was chosen as the correct choice.

Read more about Median here https://brainly.com/question/14532771

#SPJ1

Your mom Paid $8 for 10 Granny Smithapples and bought 15 Golden Delicious at60 cents an apple. What is her average costper apple?

Answers

First, let's find the total cost of all the apples:

[tex]8+(0.6\cdot15)=17[/tex]

Now, we divide this by the total amount of apples bought. In our case, we know that there were 10 Granny Smith apples and 15 Golden Delicious, for a total of 25 apples.

[tex]\frac{17}{25}\rightarrow0.68[/tex]

Therfore, we can conclude that the average cost per apple was $0.68

why is sampling with replacement used? to ensure that the sample size is as small as possible to ensure that individuals cannot be selected twice to ensure that the proportions of subgroups in the sample are exactly the same as their proportions in the population to ensure that the probability of selecting any specific individual stays constant

Answers

To determine  probability with replacement, It uses sampling with replacement. In other words, you want to determine the likelihood of an event in which you choose a ball, card, or other object from a set of options and then swap it out after each choice.

Example:

Consider a scenario in which you wished to sample two people from a population of seven.

Those people are:

John, Jack ,Qiu, Tina, Hatty, Jacques, Des

Their names could be placed in a hat. If you sample with replacement, you would pick one name, put it back in the hat, and then pick a different name. Your two-name sample has the following potential outcomes:

John, John

John, Jack

John, Qui

Jack, Qui

Jack Tina

…and so on.

The two products you sample with replacement are independent. In other words, the outcome of one has no bearing on the other. The odds of picking the first name are 1/7, and the odds of picking the second name are 1/7.

P(John, John) = (1/7) * (1/7) = .02.

P(John, Jack) = (1/7) * (1/7) = .02.

P(John, Qui) = (1/7) * (1/7) = .02.

P(Jack, Qui) = (1/7) * (1/7) = .02.

P(Jack Tina) = (1/7) * (1/7) = .02.

To learn more about probability click here:

brainly.com/question/14210034

#SPJ4

What is the slope that passes through these points?

(0, -4) and (-5, -5)

Answers

Answer:  m=1/5

Step-by-step explanation:

Which are the solutions of the equation x^4-5x^2-14=0? Use factoring to solve.

Answers

Answer:

x = ±√7  and   x=±i√2

Step-by-step explanation:

x^4-5x^2-14=0

Factor

(x^2 - 7) ( x^2 +2) =0

Using the zero product property

x^2 -7 =0     x^2 + 2 =0

x^2 =7            x^2 = -2

Taking the square root of each side

(x^2)^1/2 =7^1/2           ( x^2)^1/2 = (-2)^1/2

x = ±√7                              x=±i√2

what is t - 18 = 6 can you help me

Answers

Answer:

[tex]\boxed{\sf{t=24}}[/tex]

Step-by-step explanation:

Find the value of t, to isolate on one side of the equation.

t-18=6

First, add by 18 from both sides.

[tex]\rightarrow \sf{t-18+18=6+18}[/tex]

Solve.

Add the numbers from left to right.

6+18=24

[tex]\Rightarrow \boxed{\sf{t=24}}[/tex]

Therefore, the final answer is t=24.

The answer should have a positive sign.

I hope this helps, let me know if you have any questions.

[tex]\huge\text{Hey there!}[/tex]


[tex]\mathsf{t - 18 = 6}[/tex]

[tex]\large\text{ADD \boxed{\textsf{18}} to BOTH SIDES:}[/tex]

[tex]\mathsf{t - 18 + 18 = 6 + 18}[/tex]

[tex]\large\text{CANCEL out: \boxed{\mathsf{-18 + 18}} because it gives you 0.}[/tex]

[tex]\large\text{KEEP: \boxed{\mathsf{6 + 18}} because it gives you the answer of the t-value.}[/tex]

[tex]\large\text{New equation: }\mathsf{t = 6 + 18}[/tex]

[tex]\mathsf{t = 6 + 18}[/tex]

[tex]\mathsf{t = 24}[/tex]


[tex]\huge\text{Therefore, your answer should be: \boxed{\mathsf{t = 24}}}\huge\checkmark[/tex]


[tex]\huge\text{Good luck on your assignment \& enjoy your day!}[/tex]


~[tex]\frak{Amphitrite1040:)}[/tex]

can i gett some help pls

Answers

Answer:

26.425

Step-by-step explanation:

just tell me the slope of the line and where to plot the segments on the graph

Answers

Answer:

The slope is 2

Step-by-step explanation:

Take any two points on the line.  I am going to use the points (0,-8) and (4,0)  Points are in the form (x,y)  The y values form the two points is 0 and -8.  The x values are 4 and 0.

The slope is the change in y over the change in x.

[tex]\frac{0- -8}{4-0}[/tex] = [tex]\frac{8}{4}[/tex] = 2

Find the slope of the line.
slope =
11
rise
run
5
11
40
30
20
10
O
2
4
6
8
G
G

Answers

to get the slope of any straight line, we simply need two points off of it, let's use the ones from the picture below.

[tex](\stackrel{x_1}{0}~,~\stackrel{y_1}{0})\qquad (\stackrel{x_2}{5}~,~\stackrel{y_2}{20}) ~\hfill \stackrel{slope}{m}\implies \cfrac{\stackrel{rise} {\stackrel{y_2}{20}-\stackrel{y1}{0}}}{\underset{run} {\underset{x_2}{5}-\underset{x_1}{0}}} \implies \cfrac{ 20 }{ 5 } \implies \text{\LARGE 4}[/tex]

Evaluate cos 150° without using a calculator.Ο Α.√32B. 2O C. -1/2OD. -32

Answers

Answer:

Explanation:

Note that:

[tex]\begin{gathered} cos(A+B)=cosAcosB-sinAsinB \\ cos(150^0)=cos(90+60) \end{gathered}[/tex]

Applying the addition formula given above to cos 150:

[tex]\begin{gathered} cos(150)=cos(90)cos(60)-sin(90)sin(60) \\ \\ cos(150)=0(\frac{1}{2})-1(\frac{\sqrt{3}}{2}) \\ \\ cos(150)=0-\frac{\sqrt{3}}{2} \\ \\ cos \end{gathered}[/tex]

someone please help me please

Answers

Graph

[tex]-2x+y\ge-2[/tex]

Procedure

Determine the value for x in the equation x over 5 and 7 tenths equals 2 and 3 tenths.

Answers

The solution of x in the mathematical statement is 11.73

How to determine the value of x?

From the question, the mathematical statement is given as

x over 5 and 7 tenths equals 2 and 3 tenths.

When represented as an algebraic equation, we have

x/5.10 = 2.30

To start with, we multiply both sides of the equation by 5.10

This is represented as

5.10 * x/5.10 = 2.30 * 5.10

Evaluate the product on the left-hand side

So, we have

x = 2.30 * 5.10

Evaluate the product on the right-hand side

So, we have

x = 11.73

Hence, the value of x is 11.73

Read more about equations at

https://brainly.com/question/2972832

#SPJ1

The required simplified value of the given numeral is given as 13.11.

As given in the question, to determine the value for x in the equation x over 5 and 7 tenths equals 2 and 3 tenths.

What is the equation?

The equation is the relationship between variables and represented as y = ax + b is an example of a polynomial equation.

Here,
The statment given in the word string, first transform the equation in the mathematical inscription,
So,
x / 5.7 = 2.3
Simplifying,
x = 5.7 × 2.3
x = 13.11

Thus, the required simplified value of the given numeral is given as 13.11.

Learn more about equations here:

brainly.com/question/10413253

#SPJ1

What is the simple interest rate on an account that earned $56.25 in interest after two and one-half years on a principal balance of $300? please show work

Answers

The rate of interest on the given principal is 7.5%.

Given that, simple interest = $56.25, principal = $300 and time period = [tex]2\frac{1}{2}[/tex] years.

What is the simple interest?

Simple interest is a method to calculate the amount of interest charged on a sum at a given rate and for a given period of time.

Simple interest is calculated with the following formula: S.I. = P × R × T, where P = Principal, R = Rate of Interest in % per annum, and T = Time, usually calculated as the number of years.

Now, 56.26 = (300 × R × 2.5)/100

⇒ 5625 = 750R

⇒ R = 7.5%

Therefore, the rate of interest on the given principal is 7.5%.

To learn more about the simple interest visit:

https://brainly.com/question/25845758.

#SPJ1

X AND Y (PLEASE HELP)

Answers

Answer:

HERE YOU GO HOPE IT HELPS

Adriel is going to invest in an account paying an interest rate of 2.4% compounded monthly. How much would Adriel need to invest, to the nearest ten dollars, for the value of the account to reach $120,000 in 8 years?

Answers

The principal amount is $99,055.82.

What is Compound interest?

When you add the interest you have already earned back into your principal balance, you are earning compound interest, which increases your profits. Consider that you have $1,000 in a savings account earning 5% interest annually. If you made $50 in the first year, your new balance would be $1,050.

Given:

Amount = 120,000

r = R/100

r = 2.4/100

r = 0.024 per year,

Then, solve the equation for P

P = A / [tex](1 + r/n)^{nt}[/tex]

P = 120,000.00 /[tex](1 + 0.024/12)^{(12)(8)[/tex]

P = 120,000.00 / [tex](1 + 0.002)^{(96)[/tex]

P = $99,055.82

Hence, the principal amount is $99,055.82.

Learn more about Compound interest here:

https://brainly.com/question/14295570

#SPJ1

Other Questions
you manage a network with a single location. you want to use wsus to make sure that only approved updates are applied to all client computers. client computers should download all approved updates from a wsus server in your location. you install wsus on one server. you synchronize the list of updates on the server and approve the updates that you want applied to client computers. you check the client computers and find that only the approved updates are being applied, but updates are being downloaded from the microsoft update website, not your local wsus server. what should you do? Of the 110 settlers who arrived in may 1607, nearly 70 were dead by december. Is there anything in the ship lists that helps explain why?. a logical grouping of computers that allow computer hosts to act as if they were attached to the same broadcast domain regardless of their physical location is known as: If joey walked east for 15 2/3 meters from home. Then, he walked west for 21 3/4 meters. How far was joey from home? if covariance between two variables is near 0, it implies that group of answer choices a positive relationship exists between the variables. none of the choices. the variables are not linearly related. the variables are negatively related. the variables are strongly related. 27. Scientists have observed an increase in global temperatures over the past 100 years. Which phenomena do scientists believe contributes to the increase in temperatures? A. an increase in undersea volcanic activity B. a decrease in the distance between Earth and the Sun C. an increase in certain gases released during the use of fossil fuels D. a decrease in the amount of water on Earth due to overconsumption 14. Given: JM bisects JL JM perpendicular to KLProve: TRIANGLE JMK congruent to TRIANGLE JML 8.[/1 Points]DETAILSALEXGEOM7 9.2.012.MY NOTESASK YOUR TEACHERSuppose that the base of the hexagonal pyramid below has an area of 40.6 cm2 and that the altitude of the pyramid measures 3.7 cm. A hexagonal pyramid has base vertices labeled M, N, P, Q, R, and S. Vertex V is centered above the base.Find the volume (in cubic centimeters) of the hexagonal pyramid. (Round your answer to two decimal places.) cm3 1c. Clue 1The number has three digits.Clue 2 The number is less than 140.Clue 3 The number has 7 as a factor.Clue 4 The number is even.Clue 5 The sum of the digits of the number is less than 9. Which of the following would lead to the accumulation ground water? A. precipitationB. sublimationC.infiltration D. Non of the above I need help I need help I need help I need help I need help i need help I need help When an officer makes a vehicle stop what four things should he/she transmit to dispatch american insurance co. reviewed its customers for creditworthiness. american insurance found that a number of its customers had lower credit scores than expected. without disclosing the reason, american insurance increased the rates of insurance on new customers who had credit scores below a certain threshold. american insurance: Question 9 50 POINTS!!!!!!!Fill in the blank with the correct form of the Imperfect tense. You can copy/paste theaccent marks if needed. Nosotros ______ a OBX durante los veranos. (ir) The graph of function f is shown. The graph of an exponential function passes through (minus 0.25, 10), (0, 6), (5, minus 2) also intercepts the x-axis at 1 unit. Function g is represented by the table. x -1 0 1 2 3 g(x) 15 3 0 - 3 4 - 15 16 Which statement correctly compares the two functions? A. They have the same y-intercept and the same end behavior as x approaches . B. They have the same x-intercept but different end behavior as x approaches . C. They have the same x- and y-intercepts. D. They have different x- and y-intercepts but the same end behavior as x approaches . 48.001 to the hundredths (-1,2) and (3,32)For each of the following, find the formula for an exponential function that passes through the two points given. ANYONE WHO ANSWERS ILL GIVE BRAINLIEST TO finding the slope Gerhardt lived with his mother, even though he was in his 30s. Generally, the neighbors liked Gerhardt, but when the call went out for soldiers, somebecame annoyed that Gerhardt loudly proclaimed that he had no intention to fight. One neighbor started a rumor that Gerhardt was not loyal to theNazi party because of their stance on homosexuals. Another said that he saw Gerhardt leaving a club that was only frequented by men. Onemorning, SS officers arrived to take Gerhardt away. Without a trial, he was sent straight to one of the camps. How did the changing laws affectGerman citizens like Gerhardt?O A.OB.OC.O D.The Nuremburg Laws outlawed relationships between Aryan men and Jewish women.Section 175 allowed men to be detained on charges of homosexuality with no proof.After Kristallnacht, SS officers were allowed to imprison anyone who looked Jewish.The Aryanization Laws allowed property to be seized from anyone of Jewish descent. 8v < 40 solve the inequality for v and simplify your answer as much as possible